Last visit was: 24 Apr 2024, 17:56 It is currently 24 Apr 2024, 17:56

Close
GMAT Club Daily Prep
Thank you for using the timer - this advanced tool can estimate your performance and suggest more practice questions. We have subscribed you to Daily Prep Questions via email.

Customized
for You

we will pick new questions that match your level based on your Timer History

Track
Your Progress

every week, we’ll send you an estimated GMAT score based on your performance

Practice
Pays

we will pick new questions that match your level based on your Timer History
Not interested in getting valuable practice questions and articles delivered to your email? No problem, unsubscribe here.
Close
Request Expert Reply
Confirm Cancel
SORT BY:
Date
Tags:
Difficulty: 555-605 Levelx   Assumptionx                     
Show Tags
Hide Tags
Verbal Forum Moderator
Joined: 08 Dec 2013
Status:Greatness begins beyond your comfort zone
Posts: 2101
Own Kudos [?]: 8808 [272]
Given Kudos: 171
Location: India
Concentration: General Management, Strategy
GPA: 3.2
WE:Information Technology (Consulting)
Send PM
Most Helpful Reply
GMAT Club Verbal Expert
Joined: 13 Aug 2009
Status: GMAT/GRE/LSAT tutors
Posts: 6920
Own Kudos [?]: 63658 [58]
Given Kudos: 1773
Location: United States (CO)
GMAT 1: 780 Q51 V46
GMAT 2: 800 Q51 V51
GRE 1: Q170 V170

GRE 2: Q170 V170
Send PM
Board of Directors
Joined: 18 Jul 2015
Status:Emory Goizueta Alum
Posts: 3600
Own Kudos [?]: 5425 [5]
Given Kudos: 346
Send PM
General Discussion
Retired Moderator
Joined: 26 Nov 2012
Posts: 473
Own Kudos [?]: 493 [2]
Given Kudos: 46
Send PM
Re: Jay: Of course there are many good reasons to support the expansion of [#permalink]
1
Kudos
1
Bookmarks
Skywalker18 wrote:
Jay: Of course there are many good reasons to support the expansion of preventive medical care, but arguments claiming that it will lead to greater societal economic gains are misguided. Some of the greatest societal expenses arise from frequent urgent-care needs for people who have attained a long life due to preventive care.

Sunil: Your argument fails because you neglect economic gains outside the health care system: society suffers an economic loss when any of its productive members suffer from preventable illness.

Sunil's response to Jay makes which of the following assumptions?

A. Those who receive preventive care are not more likely to need urgent care than are those who do not receive preventive care
B. Jay intends the phrase "economic gains" to refer only to gains accruing to institutions within the health care system.
C. Productive members of society are more likely than others to suffer preventable illnesses.
D.The economic contributions of those who receive preventive medical care may outweigh the economic losses caused by preventive care.
E. Jay is incorrect in stating that patients who receive preventive medical care are long-lived.


Jay : People who receive the preventive care are arising the societal expenses.

Sunil : Society suffers an economic loss when any of its productive members suffer from preventable illness.

We can understand from this statement that as per Sunil's comment when people are healthy they'll earn more + spend more and this will in turn helps the society. This is the assumption.

Let's see the option that has is related with our paraphrase i.e link between people and economy.

A. Those who receive preventive care are not more likely to need urgent care than are those who do not receive preventive care...This option doesn't deal with people and economy.
B. Jay intends the phrase "economic gains" to refer only to gains accruing to institutions within the health care system...This option doesn't deal with people and economy.
C. Productive members of society are more likely than others to suffer preventable illnesses...This option doesn't deal with people and economy.
D.The economic contributions of those who receive preventive medical care may outweigh the economic losses caused by preventive care.
E. Jay is incorrect in stating that patients who receive preventive medical care are long-lived...This option doesn't deal with people and economy.

Here only D matches our pre-thiking.
SVP
SVP
Joined: 27 May 2012
Posts: 1680
Own Kudos [?]: 1422 [0]
Given Kudos: 632
Send PM
Re: Jay: Of course there are many good reasons to support the expansion of [#permalink]
Skywalker18 wrote:
Jay: Of course there are many good reasons to support the expansion of preventive medical care, but arguments claiming that it will lead to greater societal economic gains are misguided. Some of the greatest societal expenses arise from frequent urgent-care needs for people who have attained a long life due to preventive care.

Sunil: Your argument fails because you neglect economic gains outside the health care system: society suffers an economic loss when any of its productive members suffer from preventable illness.

Sunil's response to Jay makes which of the following assumptions?

A. Those who receive preventive care are not more likely to need urgent care than are those who do not receive preventive care
B. Jay intends the phrase "economic gains" to refer only to gains accruing to institutions within the health care system.
C. Productive members of society are more likely than others to suffer preventable illnesses.
D.The economic contributions of those who receive preventive medical care may outweigh the economic losses caused by preventive care.
E. Jay is incorrect in stating that patients who receive preventive medical care are long-lived.


It Should be D.
For productive members if their economic contributions are less than what it costs for their preventive care then Sunils argument will fall apart.
Director
Director
Joined: 02 Sep 2016
Posts: 528
Own Kudos [?]: 194 [3]
Given Kudos: 275
Re: Jay: Of course there are many good reasons to support the expansion of [#permalink]
1
Kudos
2
Bookmarks
I will give it a try..

Jay: Of course there are many good reasons to support the expansion of preventive medical care, but arguments claiming that it will lead to greater societal economic gains are misguided. Some of the greatest societal expenses arise from frequent urgent-care needs for people who have attained a long life due to preventive care.

Jay believes that expansion of preventive medical care is fine but it will not really lead to societal economic gains. Why so?
Because frequent urgent care needs will lead to greater societal expenses. So how is the society gaining?

Sunil: Your argument fails because you neglect economic gains outside the health care system: society suffers an economic loss when any of its productive members suffer from preventable illness.

Sunil thinks/believes completely opposite of what Jay believes. According to Sunil, a loss is not about money/gains but it is about the members who are productive. They fall ill and won't be contributing anything to the society. Instead will be using money for medical care.

Sunil's response to Jay makes which of the following assumptions?

A. Those who receive preventive care are not more likely to need urgent care than are those who do not receive preventive care
INCORRECT. I do not see how it kind of bridges the gap between the two statements.

B. Jay intends the phrase "economic gains" to refer only to gains accruing to institutions within the health care system.
We know that already because it was mentioned by Sunil. Also it does not really answers the question. INCORRECT

C. Productive members of society are more likely than others to suffer preventable illnesses.
But what about the economic gains and productive members of society. INCORRECT.

D.The economic contributions of those who receive preventive medical care may outweigh the economic losses caused by preventive care.
Yes!! It makes the argument more clear.

E. Jay is incorrect in stating that patients who receive preventive medical care are long-lived.
INCORRECT.
Moderator
Joined: 28 Mar 2017
Posts: 1090
Own Kudos [?]: 1970 [0]
Given Kudos: 200
Location: India
Concentration: Finance, Technology
GMAT 1: 730 Q49 V41
GPA: 4
Send PM
Re: Jay: Of course there are many good reasons to support the expansion of [#permalink]
Hello Skywalker18, Could you please post the OE for the question. I am unable to comprehend as to why option "D" is correct.

Jay: Of course there are many good reasons to support the expansion of preventive medical care, but arguments claiming that it will lead to greater societal economic gains are misguided. Some of the greatest societal expenses arise from frequent urgent-care needs for people who have attained a long life due to preventive care.

Sunil: Your argument fails because you neglect economic gains outside the health care system: society suffers an economic loss when any of its productive members suffer from preventable illness.

Sunil's response to Jay makes which of the following assumptions?

A. Those who receive preventive care are not more likely to need urgent care than are those who do not receive preventive care
B. Jay intends the phrase "economic gains" to refer only to gains accruing to institutions within the health care system.
C. Productive members of society are more likely than others to suffer preventable illnesses.
D.The economic contributions of those who receive preventive medical care may outweigh the economic losses caused by preventive care.
E. Jay is incorrect in stating that patients who receive preventive medical care are long-lived.
Intern
Intern
Joined: 17 Mar 2017
Posts: 22
Own Kudos [?]: 7 [1]
Given Kudos: 27
Concentration: Marketing, Operations
GMAT 1: 620 Q48 V28
GMAT 2: 660 Q44 V36
Send PM
Re: Jay: Of course there are many good reasons to support the expansion of [#permalink]
1
Kudos
gmatexam439 wrote:
Hello, Could you please post the OE for the question. I am unable to comprehend as to why option "D" is correct.

Jay: Of course there are many good reasons to support the expansion of preventive medical care, but arguments claiming that it will lead to greater societal economic gains are misguided. Some of the greatest societal expenses arise from frequent urgent-care needs for people who have attained a long life due to preventive care.

Sunil: Your argument fails because you neglect economic gains outside the health care system: society suffers an economic loss when any of its productive members suffer from preventable illness.

Sunil's response to Jay makes which of the following assumptions?

A. Those who receive preventive care are not more likely to need urgent care than are those who do not receive preventive care
B. Jay intends the phrase "economic gains" to refer only to gains accruing to institutions within the health care system.
C. Productive members of society are more likely than others to suffer preventable illnesses.
D.The economic contributions of those who receive preventive medical care may outweigh the economic losses caused by preventive care.
E. Jay is incorrect in stating that patients who receive preventive medical care are long-lived.



Let me try..

A. Those who receive preventive care are not more likely to need urgent care than are those who do not receive preventive care - No, even if people who receive preventive care are not more likely to need urgent care. it does not establish anything what Sunil talks about related to economic gains
B. Jay intends the phrase "economic gains" to refer only to gains accruing to institutions within the health care system. - No, What Jay says is not what the question asks.
C. Productive members of society are more likely than others to suffer preventable illnesses. - No, this is the trap here.. because it tries to link what suil says to what Jay said.. but to prove this we need to make a few more assumptions..
D.The economic contributions of those who receive preventive medical care may outweigh the economic losses caused by preventive care. - Correct.. try the negation test here.. if the economic contributions of those receiving care are less than the economic losses by providing that care.. Sunils argument falls apart.
E. Jay is incorrect in stating that patients who receive preventive medical care are long-lived - No, again.. what Jay says is not what is asked in the question

Hope it helps.. What was your answer choice?
Moderator
Joined: 28 Mar 2017
Posts: 1090
Own Kudos [?]: 1970 [2]
Given Kudos: 200
Location: India
Concentration: Finance, Technology
GMAT 1: 730 Q49 V41
GPA: 4
Send PM
Re: Jay: Of course there are many good reasons to support the expansion of [#permalink]
2
Kudos
abhimahna wrote:
gmatexam439 wrote:
Hello Skywalker18, Could you please post the OE for the question. I am unable to comprehend as to why option "D" is correct.



Hi gmatexam439 ,

Here I go:

Jay said " some arguments are misguided". On what basis did he say so " No gains to local people happening."

Sunil said : Your conclusion is wrong. Why? You didn't consider gains to outside people.

He is saying wrong. Then he must have something in his mind.

Now, consider it like this:

If gains are more, What Sunil said is valid.

If losses are more, what Sunil said is invalid.

So, when he was saying something, he must be assuming that gains are more. Hence, D is correct.

Does that make sense?


Yes bruh, that makes a lot of sense. Actually I was stuck between, B and D. I mis-read or I don't know what happened but I forgot that "economic gains" is being taken on an economy level and not just at hospital level as given in the option. Its written clearly in the premise.

Yes option D should be correct. Thank you for replying quickly.

Regards
Intern
Intern
Joined: 17 Aug 2017
Posts: 9
Own Kudos [?]: 24 [1]
Given Kudos: 4
Send PM
Re: Jay: Of course there are many good reasons to support the expansion of [#permalink]
1
Bookmarks
Skywalker18 wrote:
Jay: Of course there are many good reasons to support the expansion of preventive medical care, but arguments claiming that it will lead to greater societal economic gains are misguided. Some of the greatest societal expenses arise from frequent urgent-care needs for people who have attained a long life due to preventive care.

Sunil: Your argument fails because you neglect economic gains outside the health care system: society suffers an economic loss when any of its productive members suffer from preventable illness.

Sunil's response to Jay makes which of the following assumptions?

A. Those who receive preventive care are not more likely to need urgent care than are those who do not receive preventive care
B. Jay intends the phrase "economic gains" to refer only to gains accruing to institutions within the health care system.
C. Productive members of society are more likely than others to suffer preventable illnesses.
D.The economic contributions of those who receive preventive medical care may outweigh the economic losses caused by preventive care.
E. Jay is incorrect in stating that patients who receive preventive medical care are long-lived.



Approach:
Jay says that there are many good reasons to support preventive medical care but there are a few cases as well that demonstrates its downsides. For example, people who live longer because of preventive care may cause urgent care expenses which wouldn't be required if they were not be alive.

Sunils counters the example by saying that there are productive people whose losses might affect economic gains outside the healthcare system such as technology and innovation, etc.

Hence, Jays confined its reach(or universe) to only health care system where as Sunil puts up the argument by expanding the reach (or universe) of Jay's argument. To put it simply, people who will be saved by preventive medical care may contribute to society in other economic ways. This is what option D says.
A. Argument is not about the likeliness of people receiving medical care but about the economic contributions that those people make who receive preventive medical care.
B. Sunil's assumption has nothing to do with what Jay intends from a particular phrase such as "economic gains"
C. Same as A; not about the likeliness of who will be more prone to suffering preventable illnesses
E. Sunil never questions the correctness of Jay; just pointing out that the spectrum that Jay has considered should be broadened.

Hope this helps!
Moderator
Joined: 28 Mar 2017
Posts: 1090
Own Kudos [?]: 1970 [1]
Given Kudos: 200
Location: India
Concentration: Finance, Technology
GMAT 1: 730 Q49 V41
GPA: 4
Send PM
Re: Jay: Of course there are many good reasons to support the expansion of [#permalink]
1
Bookmarks
Jay: Of course there are many good reasons to support the expansion of preventive medical care, but arguments claiming that it will lead to greater societal economic gains are misguided. Some of the greatest societal expenses arise from frequent urgent-care needs for people who have attained a long life due to preventive care.

Sunil: Your argument fails because you neglect economic gains outside the health care system: society suffers an economic loss when any of its productive members suffer from preventable illness.

Sunil's response to Jay makes which of the following assumptions?

A. Those who receive preventive care are not more likely to need urgent care than are those who do not receive preventive care
-Sunil's argument is about economic gains. Incorrect.
B. Jay intends the phrase "economic gains" to refer only to gains accruing to institutions within the health care system.
-This is Jay's point of view. Out of scope.
C. Productive members of society are more likely than others to suffer preventable illnesses.
-This is nowhere stated or meant in the argument. Irrelevant.
D. The economic contributions of those who receive preventive medical care may outweigh the economic losses caused by preventive care.
-Correct. This states that the contribution>losses. Therefore, we can conclude that there are economic gains.
E. Jay is incorrect in stating that patients who receive preventive medical care are long-lived.
-This is Jay's point of view. Out of scope.
VP
VP
Joined: 12 Feb 2015
Posts: 1065
Own Kudos [?]: 2103 [0]
Given Kudos: 77
Send PM
Re: Jay: Of course there are many good reasons to support the expansion of [#permalink]
What is the assumption behind Sunil's response to Jay?

Sunils response:- Evaluate economic gains outside the health care system.

Jays claim: greatest societal expenses within health care system

Analysis:- What if gains outside > expenses inside the health care system e.g. the person who received preventive health care was a famous economist, prime minister or a professor?

Option D is correct as it captures this assumption:- The economic contributions of those who receive preventive medical care may outweigh the economic losses caused by preventive care.
Manager
Manager
Joined: 04 Feb 2016
Posts: 51
Own Kudos [?]: 24 [0]
Given Kudos: 4
Location: India
Concentration: Technology, Marketing
GMAT 1: 650 Q48 V32
GPA: 4
WE:Sales (Computer Software)
Send PM
Re: Jay: Of course there are many good reasons to support the expansion of [#permalink]
GMATNinja

I got this question right. However had a doubt, hence thought of asking you about the same.

Is the difference between C and D is that C is the assumption required only for argument 2 to exist and D is the assumption for argument 2 in light of argument 1?

Thanks,
GMAT Club Verbal Expert
Joined: 13 Aug 2009
Status: GMAT/GRE/LSAT tutors
Posts: 6920
Own Kudos [?]: 63658 [1]
Given Kudos: 1773
Location: United States (CO)
GMAT 1: 780 Q51 V46
GMAT 2: 800 Q51 V51
GRE 1: Q170 V170

GRE 2: Q170 V170
Send PM
Re: Jay: Of course there are many good reasons to support the expansion of [#permalink]
1
Bookmarks
Expert Reply
srinjoy1990 wrote:
GMATNinja

I got this question right. However had a doubt, hence thought of asking you about the same.

Is the difference between C and D is that C is the assumption required only for argument 2 to exist and D is the assumption for argument 2 in light of argument 1?

Let's focus on Sunil's argument:
Quote:
Your argument fails because you neglect economic gains outside the health care system: society suffers an economic loss when any of its productive members suffer from preventable illness.

This argument describes a societal economic gain made when any productive member of society suffers from preventable illness. Revisiting what I wrote earlier in this thread, the argument does depend on the relative size of that economic gain, but it doesn't depend on the relative likelihood of preventable illness contracted by productive members of society.

Quote:
C. Productive members of society are more likely than others to suffer preventable illnesses.

Choice (C) fails entirely on its own because it does not address the measurement of economic gain that Sunil has described. These members of society could be much more likely or much less likely to suffer preventable illnesses than non-productive members of society, and we still wouldn't have any indication of how much societal economic gain will change as a consequence of that likelihood. Because this assumption is not required for Sunil's argument, period, we eliminate it.

Quote:
D.The economic contributions of those who receive preventive medical care may outweigh the economic losses caused by preventive care.

Choice (D) states Sunil's unspoken logic explicitly: The gains made by productive members receiving preventative care will be greater than the losses described by Jay. That's why Sunil is confident that Jay's argument fails. Yes, this means that Sunil's conclusion is made in opposition to Jay's argument. However, the most important point is that choice (D) fully articulates Sunil's assumption regarding economic gain, while choice (C) presents information that isn't necessarily related to economic gain at all.

That's why we eliminate (C) and stick with (D).

May this explanation bring you much economic gain in the form of better GMAT performance. :)
Manager
Manager
Joined: 08 Feb 2018
Posts: 71
Own Kudos [?]: 18 [1]
Given Kudos: 100
Send PM
Re: Jay: Of course there are many good reasons to support the expansion of [#permalink]
1
Kudos
GMATNinja wrote:
VKat wrote:
Hello expert,

Could you please help me in solving this question.
How did you solve this question,whats ur approach?

I would start with Jay's conclusion, which is that "arguments claiming that [the expansion of preventive medical care] will lead to greater societal economic gains are misguided."

Why are those arguments misguided?... because "Some of the greatest societal expenses arise from frequent urgent-care needs for people who have attained a long life due to preventive care." In other words, preventive care helps many people live longer, but then those very people often have urgent-care needs. Those urgent care-needs are a great societal expense.

Sunil responds by pointing out the societal economic loss that results from a LACK of preventive medical care: "society suffers an economic loss when any of its productive members suffer from preventable illness." Sunil thus concludes that Jay's argument fails.

To summarize, Jay describes one way in which the expansion of preventive medical care will lead to a societal economic LOSS. Sunil responds by describing a societal economic GAIN (i.e. avoiding the economic loss that occurs when productive members of society suffer from preventable illnesses). So, which of the answer choices represents an assumption made in Sunil's response?

Quote:
A. Those who receive preventive care are not more likely to need urgent care than are those who do not receive preventive care

Sunil's response has nothing to do with urgent care. Instead, Sunil only talks about avoiding the economic loss that occurs when productive members of society suffer preventable illnesses. Eliminate (A).

Quote:
B. Jay intends the phrase "economic gains" to refer only to gains accruing to institutions within the health care system.

Sunil responds by discussing an economic gain outside the health care system. Thus, if "economic gains" referred only to gains accruing to institutions within the health care system, Sunil's argument would not apply! Making the assumption stated in choice (B) would actually hurt Sunil's argument, so it is certainly not a required assumption. Eliminate (B).

Quote:
C. Productive members of society are more likely than others to suffer preventable illnesses.

Sunil's argument does not require that productive members of society are more likely than others to suffer preventable illnesses. Productive members could be just as likely or even less likely to suffer preventable illnesses than others. Regardless, preventive care would help avoid the economic loss that occurs when those productive members of society DO suffer preventable illnesses. Choice (C) is not a required assumption.

Quote:
D.The economic contributions of those who receive preventive medical care may outweigh the economic losses caused by preventive care.

Remember, Sunil responds by describing a societal economic GAIN (i.e. avoiding the economic loss that occurs when productive members of society suffer from preventable illnesses). But what if that economic gain is SMALLER than the economic loss described by Jay? In other words, what if the frequent urgent-care expenses outweigh the economic gain described by Sunil? In that case, Jay's conclusion (that arguments claiming that the expansion of preventive medical care will lead to greater societal economic gains are misguided), would still be valid. Sunil is trying to argue that Jay's argument fails. Without making this assumption, Jay's argument will NOT fail, so choice (D) looks good.

Quote:
E. Jay is incorrect in stating that patients who receive preventive medical care are long-lived.

Sunil only talks about avoiding the economic loss that occurs when productive members of society suffer preventable illnesses. This argument is valid regardless of whether patients who receive preventive medical care are long-lived. Eliminate (E).

Choice (D) is the best answer.

I hope that helps!



In choice B explanation, what does it mean when you say Sunil's argument would not apply. Is it like, it wouldn't even stand to consider? How about this line of thinking: Sunil's argument that Jay's argument fails because he did not consider economic gains for which option B is an enabler.
GMAT Club Verbal Expert
Joined: 13 Aug 2009
Status: GMAT/GRE/LSAT tutors
Posts: 6920
Own Kudos [?]: 63658 [1]
Given Kudos: 1773
Location: United States (CO)
GMAT 1: 780 Q51 V46
GMAT 2: 800 Q51 V51
GRE 1: Q170 V170

GRE 2: Q170 V170
Send PM
Re: Jay: Of course there are many good reasons to support the expansion of [#permalink]
1
Kudos
Expert Reply
akshaykotha wrote:
GMATNinja wrote:
VKat wrote:
Hello expert,

Could you please help me in solving this question.
How did you solve this question,whats ur approach?

I would start with Jay's conclusion, which is that "arguments claiming that [the expansion of preventive medical care] will lead to greater societal economic gains are misguided."

Why are those arguments misguided?... because "Some of the greatest societal expenses arise from frequent urgent-care needs for people who have attained a long life due to preventive care." In other words, preventive care helps many people live longer, but then those very people often have urgent-care needs. Those urgent care-needs are a great societal expense.

Sunil responds by pointing out the societal economic loss that results from a LACK of preventive medical care: "society suffers an economic loss when any of its productive members suffer from preventable illness." Sunil thus concludes that Jay's argument fails.

To summarize, Jay describes one way in which the expansion of preventive medical care will lead to a societal economic LOSS. Sunil responds by describing a societal economic GAIN (i.e. avoiding the economic loss that occurs when productive members of society suffer from preventable illnesses). So, which of the answer choices represents an assumption made in Sunil's response?

Quote:
A. Those who receive preventive care are not more likely to need urgent care than are those who do not receive preventive care

Sunil's response has nothing to do with urgent care. Instead, Sunil only talks about avoiding the economic loss that occurs when productive members of society suffer preventable illnesses. Eliminate (A).

Quote:
B. Jay intends the phrase "economic gains" to refer only to gains accruing to institutions within the health care system.

Sunil responds by discussing an economic gain outside the health care system. Thus, if "economic gains" referred only to gains accruing to institutions within the health care system, Sunil's argument would not apply! Making the assumption stated in choice (B) would actually hurt Sunil's argument, so it is certainly not a required assumption. Eliminate (B).

Quote:
C. Productive members of society are more likely than others to suffer preventable illnesses.

Sunil's argument does not require that productive members of society are more likely than others to suffer preventable illnesses. Productive members could be just as likely or even less likely to suffer preventable illnesses than others. Regardless, preventive care would help avoid the economic loss that occurs when those productive members of society DO suffer preventable illnesses. Choice (C) is not a required assumption.

Quote:
D.The economic contributions of those who receive preventive medical care may outweigh the economic losses caused by preventive care.

Remember, Sunil responds by describing a societal economic GAIN (i.e. avoiding the economic loss that occurs when productive members of society suffer from preventable illnesses). But what if that economic gain is SMALLER than the economic loss described by Jay? In other words, what if the frequent urgent-care expenses outweigh the economic gain described by Sunil? In that case, Jay's conclusion (that arguments claiming that the expansion of preventive medical care will lead to greater societal economic gains are misguided), would still be valid. Sunil is trying to argue that Jay's argument fails. Without making this assumption, Jay's argument will NOT fail, so choice (D) looks good.

Quote:
E. Jay is incorrect in stating that patients who receive preventive medical care are long-lived.

Sunil only talks about avoiding the economic loss that occurs when productive members of society suffer preventable illnesses. This argument is valid regardless of whether patients who receive preventive medical care are long-lived. Eliminate (E).

Choice (D) is the best answer.

I hope that helps!



In choice B explanation, what does it mean when you say Sunil's argument would not apply. Is it like, it wouldn't even stand to consider? How about this line of thinking: Sunil's argument that Jay's argument fails because he did not consider economic gains for which option B is an enabler.

If Jay's definition of "economic gain" only pertains to the health care system, then Jay's conclusion doesn't need to acknowledge anything outside of the health care system.

This means that Sunil's counter-argument is totally outside the scope of Jay's argument. Sunil's statement no longer applies because it points to gains outside the health care system, while Jay is only concerned with gains inside the health care system. So if (B) is true, then there's no way that Sunil's response can cause Jay's argument to fail. They're no longer even part of the same premise.

I hope this helps!
Manager
Manager
Joined: 24 Mar 2018
Posts: 198
Own Kudos [?]: 41 [0]
Given Kudos: 288
Location: India
GMAT 1: 680 Q50 V31
Send PM
Re: Jay: Of course there are many good reasons to support the expansion of [#permalink]
gmatexam439 wrote:
abhimahna wrote:
gmatexam439 wrote:
Hello Skywalker18, Could you please post the OE for the question. I am unable to comprehend as to why option "D" is correct.



Hi gmatexam439 ,

Here I go:

Jay said " some arguments are misguided". On what basis did he say so " No gains to local people happening."

Sunil said : Your conclusion is wrong. Why? You didn't consider gains to outside people.

He is saying wrong. Then he must have something in his mind.

Now, consider it like this:

If gains are more, What Sunil said is valid.

If losses are more, what Sunil said is invalid.

So, when he was saying something, he must be assuming that gains are more. Hence, D is correct.

Does that make sense?


Yes bruh, that makes a lot of sense. Actually I was stuck between, B and D. I mis-read or I don't know what happened but I forgot that "economic gains" is being taken on an economy level and not just at hospital level as given in the option. Its written clearly in the premise.

Yes option D should be correct. Thank you for replying quickly.

Regards


gmatexam439
Can you please explain how B is wrong ?
Manager
Manager
Joined: 30 Sep 2017
Posts: 129
Own Kudos [?]: 122 [0]
Given Kudos: 658
GMAT 1: 710 Q48 V38
GPA: 3.8
Send PM
Re: Jay: Of course there are many good reasons to support the expansion of [#permalink]
Hello guys,

(D) The economic contributions of those who receive preventive medical care may outweigh the economic losses caused by preventive care.

Could anybody please explain why the word "may" isn't ambiguous in D. I eliminated D just because of that word. I felt wondering "so which one is it?": may or may not outweigh the economic losses caused by preventive care? If it does outweigh, then surely it's Sunil's assumption. Otherwise, it hurts his conclusion.

Quote:
GMATNinjaTwo
,
Quote:
gmat1393
,
Quote:
GMATNinja
,
Quote:
broall
,
Quote:
nightblade354
GMAT Club Verbal Expert
Joined: 13 Aug 2009
Status: GMAT/GRE/LSAT tutors
Posts: 6920
Own Kudos [?]: 63658 [4]
Given Kudos: 1773
Location: United States (CO)
GMAT 1: 780 Q51 V46
GMAT 2: 800 Q51 V51
GRE 1: Q170 V170

GRE 2: Q170 V170
Send PM
Re: Jay: Of course there are many good reasons to support the expansion of [#permalink]
3
Kudos
1
Bookmarks
Expert Reply
jawele wrote:
Hello guys,

(D) The economic contributions of those who receive preventive medical care may outweigh the economic losses caused by preventive care.

Could anybody please explain why the word "may" isn't ambiguous in D. I eliminated D just because of that word. I felt wondering "so which one is it?": may or may not outweigh the economic losses caused by preventive care? If it does outweigh, then surely it's Sunil's assumption. Otherwise, it hurts his conclusion.

It is ambiguous, and you're right to call attention to this word. But don't stop reasoning once you recognize that one word can have multiple meanings. The more critical question is: Is that a valid reason for rejecting the answer choice? And the answer is no.

Logically speaking, "may" and "may not" don't cancel each other out. They simply present a wide range of possible outcomes. If we say:

    Sunil assumes that the contributions may outweigh the losses,

all we're saying logically is:

    Sunil assumes that it's possible for the contributions to outweigh the losses.

This possibility doesn't guarantee or prove that the contributions will outweigh the losses. And it leaves room for the losses to outweigh the contributions. But remember, we were asked if choice (D) is being assumed by Sunil's argument. We're not looking for the choice that make Sunil's argument strong enough to beat Jay's. We're looking for the choice that Sunil's argument requires in order to be logically sound.

Does Sunil's argument require that it's possible for contributions to outweigh losses? Yes! His argument would fall apart without this basic possibility.

The English language (and the GMAT) is an incredibly frustrating diaper fire full of words that can have multiple meanings. What we're doing when we analyze individual words is figuring out how they affect the logical argument, the meaning of the sentence, or the meaning of the passage.

I hope this helps!
Intern
Intern
Joined: 27 Oct 2018
Posts: 29
Own Kudos [?]: 6 [0]
Given Kudos: 1
GMAT 1: 750 Q49 V42
GPA: 3.9
Send PM
Re: Jay: Of course there are many good reasons to support the expansion of [#permalink]
A is irrelevant, Sunil’s argument has nothing to do with whether or not those who receive preventative care are more likely to need urgent care, or not. Sunil is arguing that the loss society suffers from its productive members falling ill is greater than the cost of providing preventative care. B actually hurts Sunil’s argument. Sunil’s argument refers to costs outside the healthcare system, if this assumption is made then Sunil’s argument becomes irrelevant. C is again irrelevant, Sunil’s argument has nothing to do with whether or not productive members of society fall ill more. D is pretty good, I think it is the right answer. Sunil’s argument can be weakened by raising the question of whether or not the loss avoided through preventative care is greater than the cost of the care. This assumption confirms that the gain does, in fact, outweigh the cost. E is again irrelevant.

D is the right answer.
GMAT Club Bot
Re: Jay: Of course there are many good reasons to support the expansion of [#permalink]
 1   2   3   
Moderators:
GMAT Club Verbal Expert
6920 posts
GMAT Club Verbal Expert
238 posts
CR Forum Moderator
832 posts

Powered by phpBB © phpBB Group | Emoji artwork provided by EmojiOne